LSAT and Law School Admissions Forum

Get expert LSAT preparation and law school admissions advice from PowerScore Test Preparation.

 eober
  • Posts: 107
  • Joined: Jul 24, 2014
|
#16407
Hi,

For answer choice A, why can't we support an argument by stating that there are other instances that such an event also occurs?

Thanks!
 David Boyle
PowerScore Staff
  • PowerScore Staff
  • Posts: 836
  • Joined: Jun 07, 2013
|
#16430
eober wrote:Hi,

For answer choice A, why can't we support an argument by stating that there are other instances that such an event also occurs?

Thanks!
Hello,

Answer A may support a little, but it isn't nearly as good as answer B, which mentions "Studies consistently show".

David
 pasu1223
  • Posts: 12
  • Joined: Aug 21, 2017
|
#38760
Hello,

I got this question correct, but just wanted to double check my thought process of how to arrive at this answer.

Conclusion: "Not the case that books delivered via computer will make printed books obsolete"

P: 50/70 students printed the assignment

The LSAT speaker is assuming that her classroom's behavior is representative of the total book consumer market.

A) Should not be chosen because "several" is vague, could be 2/100 students, does not strengthen
B) Correct answer as this supports this professors claim, and shows this behavior is common not just in their class
C) Should not be selected as "some people" is vague
D) Assumes all electronic books are scanned, out of scope
E) Out of scope

Thanks!
 AthenaDalton
PowerScore Staff
  • PowerScore Staff
  • Posts: 296
  • Joined: May 02, 2017
|
#39062
Hi pasu,

Your reasoning is on point. Nicely done! :-D

Athena Dalton
 hassan66
  • Posts: 51
  • Joined: Jul 19, 2018
|
#49951
Hi,

Thank you for the above reasonings! I eliminated B because I thought most computer users was defining only a subsection of users. Yes, most computer users may print out longer reading material but what if that group was only 3 people?

I now understand how several (in A) actually means less than many or most and could mean 3 or four and how some is not strong enough because while it could mean all, it could also mean only one, D doesn't say that the "unless" portion doesn't happen and E is irrelevant. But despite seeing all the errors in reasoning in all of these, I am having trouble seeing how the subsection of people in the world that are computer users would strengthen the point. Even though it is most of the computer users, we don't know how large this population is so how can we say that books won't become obsolete? Is it that books are generally longer than a few pages so there will be at least some people in the world who will want to print materials out?

A) yes they print out reading assignments, but that doesn't mean that they'll print out books. Maybe they enjoy reading books on a screen but not shorter reading assignments
C) they could look at the screen in short intervals instead of long periods
D&E are irrelevant

B) even if computer users is a small subsect of the population, most of this group will print out reading material and they'll print out something if it is longer than a few pages which you can reasonably infer that this means they will continue wanting physical printed books so at least some people in the world will want to read printed books.

Thank you in advance!
 Adam Tyson
PowerScore Staff
  • PowerScore Staff
  • Posts: 5153
  • Joined: Apr 14, 2011
|
#50110
Your reasoning looks good, hasan66, but remember that in a Strengthen scenario we are not necessarily trying to prove that the author is correct, but only to add more evidence to support his claim. We need the answer that helps the most, relative to the other four answer choices, but even the one that we pick need not be a lot of help. Even if the "most" is just 50% + 1 person, that still helps the claim that books will not become obsolete, because more than half of this group will still print them.

If it helps, it's a Strengthen! Don't confuse this question type for a Justify the Conclusion, with its much higher standards.
 hassan66
  • Posts: 51
  • Joined: Jul 19, 2018
|
#57007
Got it! I do tend to confuse the two and end up wasting time mulling over the relative strengths of various answer choices. This is good to keep in mind!
User avatar
 PresidentLSAT
  • Posts: 87
  • Joined: Apr 19, 2021
|
#99496
One of the explanations on here says A is wrong because it requires an assumption to characterize 'several.'

Isn't this the same for B? "More than a few pages" requires assuming that 50 pages fit this characterization. How can we assume that '50 pages' is "more than a few" but not a full book?
 Robert Carroll
PowerScore Staff
  • PowerScore Staff
  • Posts: 1787
  • Joined: Dec 06, 2013
|
#100484
PresidentLSAT,

You're misunderstanding what "several" was being compared to. "Several" in answer choice (A) is compared to "most" in answer choice (B). Answer choice (B) is about what most computer users do in all situations. Answer choice (A) is about what some professors have seen some people doing. Answer choice (A) doesn't indicate anything general at all.

Robert CArroll

Get the most out of your LSAT Prep Plus subscription.

Analyze and track your performance with our Testing and Analytics Package.